The graph of y=x^3 is transformed as shown in the graph below. Which equation represents the transformed function?



y = x cubed minus 4
y = (x minus 4) cubed
y = (negative x minus 4) cubed
y = (negative x) cubed minus 4

The Graph Of Y=x^3 Is Transformed As Shown In The Graph Below. Which Equation Represents The Transformed

Answers

Answer 1

Answer:

y = (-x)^3 - 4

Step-by-step explanation:

Ok, for the function:

y = x^3

When x = 0, we have:

y = 0^3  = 0

So the original graph passes through the point (0, 0)

If we look at the given graph, we can see that the y-intercept (the value of y when x = 0) is:

y = -4

So, this is the graph of y = x^3 moved down 4 units.

You can also see that the graph goes downward as x increases (and up as x decreases) while for the function:

y = x^3

as x increases, we should see that y also increases.

Then we have a reflection across the x-axis.

Ok, now let's describe a vertical shift.

For a general function f(x), a vertical shift of N units is written as:

g(x) = f(x) + N

if N is positive, the shift is upwards

if N is negative, the shift is downwards.

And for a function f(x), a reflection across the x-axis is written as:

g(x) = - f(x)

Here we first apply the reflection across the x-axis, so we get:

g(x) = -f(x)

now we apply the shift 4 units downwards

g(x) = - f(x) - 4

replacing f(x) by our function, x^3

we get:

g(x) = -x^3 - 4

And because of the odd power, we can write:

-x^3 = (-x)^3

Then the function is:

g(x) = (-x)^3 - 4

The correct option is the last one.

y = (-x)^3 - 4


Related Questions

Get brainly if right!! Plsss help

A dozen roses in a gift box cost 21 dollars. Twenty roses in a
gift box cost 32.6 dollars. How much does a gift box cost? How
much does one rose cost?

Answers

Answer: [tex]\$1.45[/tex]; [tex]\$3.6[/tex]

Step-by-step explanation:

Given

A dozen roses in a gift box costs $21

Similarly, 20 roses in a gift box costs $32.6

Suppose the price of single rose and gift box is [tex]x[/tex] and [tex]y[/tex]

[tex]\therefore 12x+y=21\quad \ldots(i)\\\Rightarrow 20x+y=32.6\quad \ldots(ii)[/tex]

Solving (i) and (ii) ,  we get

[tex]x=\$1.45;y=\$3.6[/tex]

Thus, the price of the one rose is [tex]\$1.45[/tex] and the price of gift box is [tex]\$3.6[/tex]

Answer:

1.45 and 3.5

Step-by-step explanation:

7. What is given in the problem?
A. Radius of 80m C. Radius of 80 ft.
B. Diameter of 40 ft. D. Diameter of 40 m paki sagot

Answers

Answer:

radius of 80cm is the answer

The asymptote of the function f(x) = 3x + 1 – 2 is . Its y-intercept is

Answers

Answer:

-1

Step-by-step explanation:

1-2=-1

y=mx+b

b= y intercept

Answer:

-1

Step-by-step explanation:

Help help help !!!!

Answers

the third choice

Step-by-step explanation:

check the exchange between logarithms and exponential functions srry but cannot write it here with my phone


If ABC is reflected across the y-axis, what are the coordinates of C?

A. (-8, -4)
B. (8,-4)
C. (-8,4)
D. (4,-8)

Answers

Answer:

c....................

I would go with c seems the most accurate one

The sum of the first ten terms of an arithmetic progression consisting of

positive integer terms is equal to the sum of the 20th, 21st and 22nd term.

If the first term is less than 20, find how many terms are required to give

a sum of 960.

Answers

Answer: [tex]n=13[/tex]

Step-by-step explanation:

Given

Sum of the first 10 terms is equal to sum of 20, 21, and 22 term

[tex]\Rightarrow \dfrac{10}{2}[2a+(10-1)d]=[a+19d]+[a+20d]+[a+21d]\\\\\Rightarrow 5[2a+9d]=3a+60d\\\Rightarrow 10a+45d=3a+60d\\\Rightarrow 7a=15d[/tex]

No of terms to give a sum of 960

[tex]\Rightarrow 960=\dfrac{n}{2}[2a+(n-1)d]\\\\\Rightarrow 1920=n[2a+(n-1)\cdot \dfrac{7}{15}a]\\\\\Rightarrow 28,800=n[30a+7a(n-1)]\\\\\Rightarrow a=\dfrac{28,800}{n[30+7n-7]}\\\\\Rightarrow a=\dfrac{28,800}{n[23+7n]}[/tex]

Value of first term is less than 20

[tex]\therefore \dfrac{28,800}{n[23+7n]}<20\\\\\Rightarrow 28,800<20n[23+7n]\\\Rightarrow 0<460n+140n^2-28,800\\\Rightarrow 140n^2+460n-28,800>0\\\\\Rightarrow n>12.79\\\\\text{For integer value }\\\Rightarrow n=13[/tex]

Answer:

15

Step-by-step explanation:

In the previous answer halfway through they used the equation: 960 = (n÷2)×(2a+(n-1)×(7a÷15))

Using this equation we can substitute an number to replace n, the higher the number is the smaller a would be.

When we substitute 15 into a, then it leaves us with the answer to be a = 15 which is a positive integer and also is smaller than 20, this then let’s us know that 15 is how many terms can be summed up to make 960.

To double check this answer you can find that d = 7 by changing the a into 15 in the formula 7a/15 (found in the previous answer.

Then in the expression: (n÷2)×(2a+(n-1)×d)

substitute:

n = 14 (must be an even number for the equation to work)

a = 15

d = 7

This will give you an answer of 847, but this is only 14 terms as we changed n into 14. To add the final term you need to complete the following equation: 847+(a+(n-1)×d)

substituting:

n = 15

a = 15

d = 7

This will give you the answer of 960, again proving that it takes 15 terms to sum together to make the number 960.

I hope this has helped you.

P.S. Everything in the previous solution was right apart from the start of the last section and the answer

Find the area of the irregular figure. Round to the nearest hundredth.

Answers

Answer:

[tex]67.5\text{ [square units]}[/tex]

Step-by-step explanation:

The composite figure consists of one rectangle and two triangles. We can add up the area of these individual shapes to find the total area of the irregular figure.

Formulas:

Area of rectangle with base [tex]b[/tex] and height [tex]h[/tex]: [tex]A=bh[/tex] Area of triangle with base [tex]b[/tex] and height [tex]h[/tex]: [tex]A=\frac{1}{2}bh[/tex]

By definition, the base and height must intersect at a 90 degree angle.

The rectangle has a base of 10 and a height of 5. Therefore, its area is [tex]A=10\cdot 5=50[/tex].

The smaller triangle to the left of the rectangle has a base of 2 and a height of 5. Therefore, its area is [tex]A=\frac{1}{2}\cdot 2\cdot 5=5[/tex].

Finally, the larger triangle on top of the rectangle has a base of 5 and a height of 5. Therefore, its area is [tex]A=\frac{1}{2}\cdot 5\cdot 5=12.5[/tex].

Thus, the area of the total irregular figure is:

[tex]50+5+12.5=\boxed{67.5\text{ [square units]}}[/tex]

Find the sum of the complex numbers (3+3i)+(8+7i)

Answers

Answer:

11 + 10i

Step-by-step explanation:

Just treat i like any other variable, and combine like terms. Hope that helps!

Ivan is playing a skee-ball game. Different points are awarded depending on which hole the ball goes through. When the ball goes in the smallest hole, it is worth 100 points. When it goes in the bigger hole, it is worth 10 points, and when it does not go in either hole, it is worth 1 point. Ivan earned 352 points in the last game.

Which combination will result in a score greater than his current score?

2 balls in the smallest hole, and 8 balls in the bigger hole
4 balls in the smallest hole, and 6 balls in neither hole
3 balls in the smallest hole, 4 balls in the bigger hole, and 3 balls in neither hole
3 balls in the smallest hole, 3 balls in the bigger hole, and 4 balls in neither hole

Answers

Answer:

B.

Step-by-step explanation:

I don't know for a fact but i think its B. Sorry if I got it wrong.

2+8+5+9+90=
3+45+111=​

Answers

Answer:

1) 114

2) 159

Step-by-step explanation:

2+8+5+9+90 = 114

3+45+111 = 159

Hope this helps.

Answer:

1)144

2)159

..........

The ratio of the number of cherry tomatoes in a tossed salad to people served is 7:15. If Waldo wants to serve 105 people, how many cherry tomatoes will Waldo use

Answers

49 tomatoes bc 7/15 is equivalent to 49/105

A statistician calculates that 8% of Americans own a Rolls Royce. If the statistician is right, what is the probability that the proportion of Rolls Royce owners in a sample of 595 Americans would differ from the population proportion by more than 3%

Answers

Answer:

0.007 = 0.7% probability that the proportion of Rolls Royce owners in a sample of 595 Americans would differ from the population proportion by more than 3%

Step-by-step explanation:

To solve this question, we need to understand the normal probability distribution and the central limit theorem.

Normal Probability Distribution:

Problems of normal distributions can be solved using the z-score formula.

In a set with mean [tex]\mu[/tex] and standard deviation [tex]\sigma[/tex], the z-score of a measure X is given by:

[tex]Z = \frac{X - \mu}{\sigma}[/tex]

The Z-score measures how many standard deviations the measure is from the mean. After finding the Z-score, we look at the z-score table and find the p-value associated with this z-score. This p-value is the probability that the value of the measure is smaller than X, that is, the percentile of X. Subtracting 1 by the p-value, we get the probability that the value of the measure is greater than X.

Central Limit Theorem

The Central Limit Theorem establishes that, for a normally distributed random variable X, with mean [tex]\mu[/tex] and standard deviation [tex]\sigma[/tex], the sampling distribution of the sample means with size n can be approximated to a normal distribution with mean [tex]\mu[/tex] and standard deviation [tex]s = \frac{\sigma}{\sqrt{n}}[/tex].

For a skewed variable, the Central Limit Theorem can also be applied, as long as n is at least 30.

For a proportion p in a sample of size n, the sampling distribution of the sample proportion will be approximately normal with mean [tex]\mu = p[/tex] and standard deviation [tex]s = \sqrt{\frac{p(1-p)}{n}}[/tex]

A statistician calculates that 8% of Americans own a Rolls Royce.

This means that [tex]p = 0.08[/tex]

Sample of 595:

This means that [tex]n = 595[/tex]

Mean and standard deviation:

[tex]\mu = p = 0.08[/tex]

[tex]s = \sqrt{\frac{p(1-p)}{n}} = \sqrt{\frac{0.08*0.92}{595}} = 0.0111[/tex]

What is the probability that the proportion of Rolls Royce owners in a sample of 595 Americans would differ from the population proportion by more than 3%?

Proportion above 8% + 3% = 11% or below 8% - 3% = 5%. Since the normal distribution is symmetric, these probabilities are equal, and so we find one of them and multiply by 2.

Probability the proportion is less than 5%:

P-value of Z when X = 0.05. So

[tex]Z = \frac{X - \mu}{\sigma}[/tex]

By the Central Limit Theorem

[tex]Z = \frac{X - \mu}{s}[/tex]

[tex]Z = \frac{0.05 - 0.08}{0.0111}[/tex]

[tex]Z = -2.7[/tex]

[tex]Z = -2.7[/tex] has a p-value of 0.0035

2*0.0035 = 0.0070

0.007 = 0.7% probability that the proportion of Rolls Royce owners in a sample of 595 Americans would differ from the population proportion by more than 3%

Which of the following is true?
1. The scale factor is 5/2 with a center of dilation at point B. The image of AB is on the same line because it passes through the center of dilation and CD is parallel to its image because it does not pass through the center of dilation.

2. The scale factor is 5/2 with a center of dilation at point B. The image of AB is on the same line because it does not pass through the center of dilation and CD is parallel to its image because it does pass through the center of dilation.

3. The scale factor is 2/5 with a center of dilation at point B. The image of AB is on the same line because it does not pass through the center of dilation and CD is parallel to its image because it does pass through the center of dilation.

4. The scale factor is 2/5 with a center of dilation at point B. The image of AB is on the same line because it passes through the center of dilation and CD is parallel to its image because it does not pass through the center of dilation.

Answers

Answer:

Option A

Step-by-step explanation:

If the quadrilateral ABCD is dilated by a scale factor 'k' to form quadrilateral A'B'C'D',

Scale factor = [tex]\frac{\text{Length of one side of the Image}}{\text{Length of one side of the original}}[/tex]

k = [tex]\frac{BA'}{BA}[/tex]

Distance between B(2, -5) and A(-1, -1) = [tex]\sqrt{(x_2-x_1)^2+(y_2-y_1)^2}[/tex]

                                                               = [tex]\sqrt{(2+1)^2+(-5+1)^2}[/tex]

                                                               = 5 units

Distance between B(2, -5) and A'(-5.5, 5) = [tex]\sqrt{(-5.5-2)^2+(5+5)^2}[/tex]

                                                                    = [tex]\sqrt{(-7.5)^2+(10)^2}[/tex]

                                                                    = 12.5 units

Scale factor 'k' = [tex]\frac{12.5}{5}[/tex]

k = [tex]\frac{5}{2}[/tex]

Therefore, ABCD is dilated by a scale factor [tex]\frac{5}{2}[/tex] about point B.

BA and it's image BA' are on the same line and passes through center of dilation B.

Similarly, lines CD and C'D' will be parallel because they do not pass through center of dilation.

Therefore, Option (A) will be the correct option.

Carin opened a money market account with a
deposit of $3,000. This account earns 2% simple
interest annually. How many years will it take for
her $3,000 deposit to earn $420 in interest, assum-
ing she does not withdraw any of the money?

Answers

Answer:

7

Step-by-step explanation:

For simple interest,

I = prt

where I = interest,

p = principal (amount deposited)

r = annual rate of interest

t = time in years

We have r = 2% = 0.02

p = $3,000

I = $420

We need to find t

I = prt

420 = 3000 * 0.02 * t

420 = 60t

t = 420/60

t = 7

Answer: 7 years

Someone pls help me due in 30 min. Given that x and y show inverse variation, complete the table.

Answers

Answer:

1st blank=[tex]y_{1}[/tex]

2nd blank=[tex]x_{1}[/tex]

3rd blank= [tex]y_{2}[/tex]

3*27=81

so 1*[tex]y_{1}[/tex]=81

hence [tex]y_{1}[/tex]=81

9* [tex]x_{1}[/tex]= 81

[tex]x_{1}[/tex]=9

27*[tex]y_{2}[/tex]=81

[tex]y_{2}[/tex]=3

Thus solved.

Hope this helps.

Please mark me as brainliest.

When you buy the latest iPad for $799, they offer you an extended warranty for $90; they will replace the iPad if it breaks in the next 2 years. The probability that it will break is 0.1. Assume there is no chance the ipad can break more than one time in the next two years. Calculate the total expected cost of the ipad if you take the warranty (i.e. include cost of ipad and warranty). Enter this value below in dollars (calculated to nearest cent, i.e. $10.90, etc).

Answers

Answer:

889 dollars

Step-by-step explanation:

State sales tax y is directly proportional to retail price x. An item that sells for 156 dollars has a sales tax of 14.42 dollars. Find a mathematical model that gives the amount of sales tax y in terms of the retail price x .

What is the sales tax on a 320 dollars purchase.

Answers

Answer:

The sales tax on a 320 dollars purchase is of $29.6.

Step-by-step explanation:

State sales tax y is directly proportional to retail price x.

This means that:

[tex]y = cx[/tex]

In which c is the constant of proportionality.

An item that sells for 156 dollars has a sales tax of 14.42 dollars.

This means that [tex]x = 156, y = 14.42[/tex]. We use this to find c. So

[tex]y = cx[/tex]

[tex]14.42 = 156c[/tex]

[tex]c = \frac{14.42}{156}[/tex]

[tex]c = 0.0924[/tex]

Then

[tex]y = 0.0924x[/tex]

What is the sales tax on a 320 dollars purchase?

y when [tex]x = 320[/tex]. So

[tex]y = 0.0924(320) = 29.6[/tex]

The sales tax on a 320 dollars purchase is of $29.6.

Jason wants to fill a cylindrical water tank to its full capacity. He knows that the volume of the tank is equal to the product of , the square of the radius of the tank, and the height of the tank. Jason measured the height of the tank and found it to be 15 feet. He also measured the radius of the cylindrical tank and found it to be 10 feet. If Vrepresents the volume of the cylindrical tank, then which of the following equations can be used to calculate the volume of the tank? ​

Answers

Answer:

B)      [tex]\sqrt{\frac{v}{15\pi } } = 10[/tex]

Step-by-step explanation:

Question 3
Solve In(x + 1) = 1.
A) X= 2
B) x = e + 1
C)x= e
D)x= e-1

Answers

Answer:

D) x = e - 1

General Formulas and Concepts:

Pre-Algebra

Equality Properties

Algebra II

Natural Logarithms ln and Euler's number eSolving logarithmic equations

Step-by-step explanation:

Step 1: Define

Identify

[tex]\displaystyle ln(x + 1) = 1[/tex]

Step 2: Solve for x

[Equality Property] e both sides:                                                                     [tex]\displaystyle e^{ln(x + 1)} = e^1[/tex]Simplify:                                                                                                             [tex]\displaystyle x + 1 = e^1[/tex][Equality Property] Isolate x:                                                                            [tex]\displaystyle x = e - 1[/tex]

A small radio transmitter broadcasts in a 69 mile radius. If you drive along a straight line from a city 93 miles north of the transmitter to a second city 78 miles east of the transmitter, during how much of the drive will you pick up a signal from the transmitter?

Answers

Answer:

See Explanation

Step-by-step explanation:

According to the Question,

We have A small radio transmitter that broadcasts in a 69-mile radius. If you drive along a straight line from a city 93 miles north of the transmitter to a second city 78 miles east of the transmitter.

Thus,

The distance that you get reception is the length of the chord created by the intersection of the circle defining the edge of transmission and the line defining the car trip.

 x2 + y2 = 69²               this is the circle

And,

The Transmitter at the origin

City to the north at (0,93)  & City to the east at (78,0)  

the Slope is M=(-93/78)

Intercept is B= y - mx ⇒  93 - (-93/78)(0) = 93

The equation of the line between the cities is y = (-93/78)x + 93

y = -93x/78 + 93           this is the line

Now, Solve the above two Equations

The intersection is gotten from the picture or solving:

x^2 + [(-93/78)*x + 93]^2 = 69^2

on solving we get, the points approximately are: (67.952,11.98 ) and (23.6277, 64.82)

Now,

From the Pythagorean theorem the total distance of the trip is:  

d1 = √(93^2 + 78^2) ≈ 121.37miles  

And the distance when the signal is picked up is:

d2 =√ [(67.952-23.627)^2 + (64.82 - 11.98)^2] ≈ 68.96 miles

You will pick up a signal from the transmitter in (d2/d1)*100 = 56% of the drive.

A table is on sale for $247, which is 76% of the regular price.
What is the regular price?

Answers

Answer:

$325

Step-by-step explanation:

Find the regular price by dividing 247 by 0.76:

247/0.76:

= 325

So, the regular price was $325

What are the solutions to the system of equations graphed below

Answers

Yes I should be about 50!593

Graph the solution for the following linear inequality system. Click on the graph until the final result is displayed.
X +3 SO
X-220
y

Answers

Answer:

Step-by-step explanation:

X+3 SO = Cymath can't further simplify this.

Please try another operation.

X-220= Cymath can't further simplify this.

Please try another operation.

y= AsymptotesFind the vertical, horizontal and slant asymptotes.

"Asymptotes y=x^2/(x+8)"

"Asymptotes y=1/x"

DifferentiateFind the derivative.

"Differentiate cos(x)^4"

"Differentiate x^5/y for x"

DomainFind the domain of a function.

"Domain y=2/x"

"Domain y=sqrt(x-3)"

The house-numbers on a certain street go from 1 to 88. The function B(n) models the type of the building whose number is n according to the following key:

(GRAPH ATTATCHED)

What number type is more appropriate for the domain of B?
A. Integer
B. Real Number

What's the appropriate domain?

Answers

Hello,

Answer A

[tex]dom (B(n)) =\{0,1,2,3\} =\{ z\ in \ \mathbb{Z} \ |\ 0 \leq z \leq 4\}[/tex]

Help plz I just need the awnser to this question

Answers

Answer:

A seems to be correct

Step-by-step explanation:

When randomly selecting​ adults, let M denote the event of randomly selecting a male and let B denote the event of randomly selecting someone with blue eyes. What does P(M|B) ​represent? Is P(M|B) the same as P(B|M)​?

Answers

Answer:

See explanation

Step-by-step explanation:

Given

[tex]M \to[/tex] randomly selecting a male

[tex]B \to[/tex] randomly selecting someone with blue eyes

Solving (a): Interpret P(M|B)

The above implies conditional probability

The interpretation is: the probability of selecting a male provided that a person with blue eyes has been selected

Solving (b): is (a) the same as P(B|M)

No, they are not the same.

The interpretation of P(B|M) is: the probability of selecting a person with blue eyes provided that a male has been selected

nen,
Problem: Two towns, A and B, located along the coast of the Pacific Ocean are 30
km apart on a north-south line. From a ship, the line of sight of town A is W30°N,
while that of town B is S400W.
1. How far is the ship from town A?
2. How far is the ship from town B?

Answers

Answer:

Step-by-step explanation:

From the picture attached,

m∠COB = 90° - m∠BOS

              = 90° - 40°

              = 50°

tan(30°) = [tex]\frac{AC}{OC}[/tex]

[tex]\frac{1}{\sqrt{3}}=\frac{AC}{OC}[/tex]

AC = [tex]\frac{OC}{\sqrt{3}}[/tex]  ------(1)

Similarly, tan(50°) = [tex]\frac{BC}{OC}[/tex]

BC = OC[tan(50°)] -------(2)

Now AC + BC = 30 cm

By substituting the values of AC and BC from equation (1) and (2),

[tex]\frac{OC}{\sqrt{3}}+OC(\text{tan}50)=30[/tex]

(1.769)OC = 30

OC = 16.96

1). cos(30°) = [tex]\frac{OC}{AO}[/tex]

[tex]\frac{\sqrt{3}}{2}= \frac{16.96}{OA}[/tex]

[tex]OA=19.58[/tex] cm

Therefore, distance between the ship and town A is 19.58 cm.

2). cos(50°) = [tex]\frac{OC}{OB}[/tex]

0.6428 = [tex]\frac{16.96}{OB}[/tex]

OB = 26.38 cm

Therefore, distance between the ship and town B is 26.38 cm.

Find the union {6, 11, 15} U Ø​

Answers

Answer:  {6, 11, 15}

Explanation:

The Ø​ means "empty set". It's the set with nothing inside it, not even 0.

We can write Ø​ as { } which is a pair of curly braces with nothing between them.

The rule is that if we union any set A with Ø​, then we'll get set A

A U Ø​ = A

Ø​ U A = A

In a sense, it's analogous to adding 0. So it's like saying A+0 = A and 0+A = A.

So that's why {6, 11, 15} U Ø​ = {6, 11, 15}

There's nothing to add onto the set {6, 11, 15}, so we just get the same thing back again.

ASK YOUR TEACHER A 12-sided die can be made from a geometric solid called dodecahedron. Assume that a fair dodecahedron is rolled. (a) What is the probability of getting a number less than 10 on a single roll

Answers

9514 1404 393

Answer:

  3/4

Step-by-step explanation:

Assuming the faces are numbered 1 to 12, there are 9 faces with values less than 10. Since the outcomes are equally probable and mutually exclusive, the probability of any of the 9 is the sum of their individual probabilities:

  P(n < 10) = 9×1/12 = 9/12 = 3/4

Find the volume (in cubic yards) of a cylinder with radius 1.2 yards and height 2.9 yards. (Round your answer to one decimal place.)

Answers

Answer:

11.8 yd³

Step-by-step explanation:

Other Questions
Find the area of the triangle with vertices A(-3,2), B(1,-2), and c(1,3) What is the dependent variable in thisexperiment?DONEBiologists designed an experiment to testthe effect of compost on the developmentof root crops. They tested several differentcrops, including carrots, potatoes, beets,and onions. They grew most of the plantsin the greenhouse, but due to space issues,they had to grow some outdoors. They gaveall the plants the same amount of compost.They obtained the compost from a localfarmer and from the local hardware store.They ran out of the farmer's compost, sosome of the plants received that compostwhen the seeds were planted and otherplants got hardware store compost afterthe plants had already started growing.What is the independent variable in thisexperiment?DONE Computer data that is suitable for text Calculate the average atomic mass for element X Using the formula for the area of a triangle, , write an expression for the area of ABC. Base your answer on the work you did in parts G through I. Show your work. PLEASE HELP NEED HELP W ALGEBRA 2 QUESTION!!the graph of f(x) = 3 sqr root x+8 is shown which statement is true 1. The function is only increasing x > -82. The function is only increasing when x > 03. The function is always decreasing4. The function is always increasingPls say what number choice in your answer! Thanks! pls anyone help mates Find the magnitude and direction of a force between a 25.0 coulomb charge and a 40.0coulomb charge when they are separated by a distance of 30.0cm Help me please I really need this tomorrow In a double-slit experiment, the slit spacing is 0.120 mm and the screen is 2.00 m from the slits. Find the wavelength (in nm) if the distance between the central bright region and the third bright fringe on a screen is 2.75 cm. Find the x of a triangle. NEED HELP ASAP. Please show all work. A point on a rotating wheel (thin hoop) having a constant angular velocity of 200 rev/min, the wheel has a radius of 1.2 m and a mass of 30 kg. ( I = mr2 ).(a) (5 points) Determine the linear acceleration.(b) (4 points) At this given angular velocity, what is the rotational kinetic energy? Which rule describes the transformation shown?1. (x,y) (-y, x+7)2. (x,y) (x+7,-y)3. (x,y) (-x, y+7)4. (x,y) (y+7, -x) How did the development of the telegraph lead to major social changesduring the Industrial Revolution?A. It forced people to learn Morse code in order to communicateacross long distances.B. It led to an era of peace as communication between differentcountries increased.C. It allowed for faster communication and required massivenetworks of cables to be laid.D. It led to a great economic inequality between those who couldafford to use it and those who couldn't. A board is 87 cm in length and must be cut so that one piece is 21 cm longer than the other piece. Find the length of each piece. Round your answers to the nearest centimeter, if necessary. * + 21? What is the Open System Foundation? Rick Co. had 36 million shares of $1 par common stock outstanding at January 1, 2021. In October 2021, Rick Co.'s Board of Directors declared and distributed a 1% common stock dividend when the market value of its common stock was $56 per share. In recording this transaction, Rick would: which of the following structures helps our sense of balance You also offer to pick her up a cup of coffee during the next break. This is a soft influence tactic, known as What is the y-intercept of the graph of y = 2.5x? a. 2.5 c. 0 b. 1 d. -1